Những câu hỏi liên quan
Trần Đông Dun
Xem chi tiết
Linh Hồ
Xem chi tiết
Anh Trâm
Xem chi tiết
Rồng Đom Đóm
22 tháng 3 2019 lúc 20:14

Ta có:\(P=a^2+\frac{1}{a^2}+b^2+\frac{1}{b^2}+c^2+\frac{1}{c^2}\)

\(\Rightarrow P\ge a^2+b^2+c^2+\frac{9}{a^2+b^2+c^2}\)(bđt cauchy-schwarz)

\(P\ge\frac{a^2+b^2+c^2}{81}+\frac{9}{a^2+b^2+c^2}+\frac{80\left(a^2+b^2+c^2\right)}{81}\)

\(\Rightarrow P\ge\frac{2}{3}+\frac{80\left(a^2+b^2+c^2\right)}{81}\left(AM-GM\right)\)

Sử dụng đánh giá quen thuộc:\(a^2+b^2+c^2\ge\frac{\left(a+b+c\right)^2}{3}=27\)

\(\Rightarrow P\ge\frac{2}{3}+\frac{80\cdot27}{81}=\frac{82}{3}\)

"="<=>a=b=c=3

nguyen kim chi
Xem chi tiết
nguyen kim chi
Xem chi tiết
huyen vu thi
Xem chi tiết
Nguyễn Phương Thảo
29 tháng 3 2016 lúc 21:23

bài này chứng minh bài toán phụ, khá là phức tạp, trình bày ra chắc chết quá

Lạy quan công đừng đánh...
29 tháng 3 2016 lúc 21:40

bài này mình thấy tren mạng đăng lên đó, có kết quả nhưng ko copy được

Phước Nguyễn
30 tháng 3 2016 lúc 9:16

Bài này bạn xem lại trong chtt ấy! Mình giải bài này rồi, giải bằng miệng cho nhanh.

đoàn danh dũng
Xem chi tiết
Phạm Thị Bắc
Xem chi tiết
Oo Bản tình ca ác quỷ oO
15 tháng 3 2016 lúc 8:16

???? là sao vừa lớn vừa bằng đó

duyệt đi

Loi Dinh
Xem chi tiết
Phước Nguyễn
10 tháng 1 2016 lúc 19:57

Theo giả thiết, ta có:

\(\left(a+b+c\right)^2=a^2+b^2+c^2\)

\(\Leftrightarrow\)  \(a^2+b^2+c^2+2\left(ab+bc+ac\right)=a^2+b^2+c^2\)

\(\Rightarrow\)  \(2\left(ab+bc+ac\right)=0\)

\(\Rightarrow\)  \(ab+bc+ac=0\)

Vì   \(a,b,c\ne0\)  nên  \(\frac{ab+bc+ac}{abc}=0\), tức là  \(\frac{1}{a}+\frac{1}{b}+\frac{1}{c}=0\)  \(\left(1\right)\)

Từ  \(\left(1\right)\)  \(\Rightarrow\)  \(\frac{1}{a}+\frac{1}{b}=-\frac{1}{c}\)  \(\left(2\right)\)

               \(\Leftrightarrow\)   \(\left(\frac{1}{a}+\frac{1}{b}\right)^3=\left(-\frac{1}{c}\right)^3\)

               \(\Leftrightarrow\)  \(\frac{1}{a^3}+\frac{1}{b^3}+3.\frac{1}{a}.\frac{1}{b}\left(\frac{1}{a}+\frac{1}{b}\right)=-\frac{1}{c^3}\)

               \(\Leftrightarrow\)  \(\frac{1}{a^3}+\frac{1}{b^3}+\frac{1}{c^3}=-\frac{3}{ab}\left(\frac{1}{a}+\frac{1}{b}\right)\)

               \(\Leftrightarrow\)   \(\frac{1}{a^3}+\frac{1}{b^3}+\frac{1}{c^3}=\frac{3}{abc}\)  (do  \(\left(2\right)\) )